X



トップページ物理
1002コメント353KB
慣性の法則は運動方程式の系だろ
レス数が1000を超えています。これ以上書き込みはできません。
0001ご冗談でしょう?名無しさん
垢版 |
2021/08/17(火) 01:24:31.33ID:???
運動方程式

ma = F

において、F = 0とおけば、m ≠ 0ならば、a = 0。
したがって、力が働いていなければ、物体は等速直線運動をする。
0953ご冗談でしょう?名無しさん
垢版 |
2021/12/06(月) 13:23:24.28ID:???
>>950
分かってないのはお前
運動方程式は慣性系でしか成り立たない
慣性系を規定しているのが第一法則
お前の質問は理論的に意味が無い
0954ご冗談でしょう?名無しさん
垢版 |
2021/12/06(月) 13:30:28.10ID:???
>>953
誰も慣性系の存在が運動方程式から導かれるなんて言っていない
慣性系で考えた場合

ma = F

にF = 0を代入したら

a = 0

か?分からないのか??
0955ご冗談でしょう?名無しさん
垢版 |
2021/12/06(月) 13:32:27.45ID:???
>>954
お前は物理が理解できてないからそんなことを気にしてるんだよ
物理が理解できてる奴は、そんなことが気にならないからもっと意味のある問題を考える
0959ご冗談でしょう?名無しさん
垢版 |
2021/12/06(月) 14:25:24.02ID:???
置いてあるボールを回転してる椅子の人から観測すれば
ボールは力がゼロなのに加速度運動する。
ma=0 a=0 は間違いである。

つまり
第一法則の慣性系から観測を前提条件にしないで、第二法則 ma=F は成り立たない。
これが理解できない人は特殊相対性理論の双子のパラドックスも理解不能。
0960ご冗談でしょう?名無しさん
垢版 |
2021/12/06(月) 14:37:19.62ID:???
>>959
日本の義務教育の物理ではそのように正確に教えていない

摩擦がなければ等速直線運動をする?とかで誤魔化してる
馬鹿レベルに合わせた悪平等教育の結果がこれだ。
0961ご冗談でしょう?名無しさん
垢版 |
2021/12/06(月) 15:31:20.47ID:???
>>959
何度も言っているように、そんなことは問題にしていない

ma = F

という方程式において、F = 0とおいたら

a = 0

か? いい加減答えろや
0962ご冗談でしょう?名無しさん
垢版 |
2021/12/06(月) 15:32:58.98ID:???
>>961
慣性系でなければ「ma = F」という式は成り立たない
回転する座標系から静止する物体を見れば、加速度運動しているように見える
F = 0だがa = 0ではない
0965ご冗談でしょう?名無しさん
垢版 |
2021/12/06(月) 15:36:19.11ID:???
>>964
そんなことは聞いていない
質問はこう

慣性系において ma = F が成り立つ。
この系において、F = 0 としたら a = 0か?

いい加減、誤魔化さないで答えてくれ
0967ご冗談でしょう?名無しさん
垢版 |
2021/12/06(月) 15:40:38.72ID:???
>>966
誰もそんなことは質問していない

物体を観測する座標系として慣性系を取ったとき、
運動方程式ma = Fが成り立つことが知られている。
さて、この座標系において、F = 0ならばa = 0か?

いい加減に答えてくれ
0968ご冗談でしょう?名無しさん
垢版 |
2021/12/06(月) 15:42:02.52ID:???
>>967
だから「ma=F」が成り立つのは慣性系においてのこと
第一法則は慣性系の存在を主張しているのであって、第二法則から導かれるのではない
0971ご冗談でしょう?名無しさん
垢版 |
2021/12/06(月) 15:48:10.08ID:???
>>970
日本語読めないのかな
誰も「慣性系の存在が運動方程式の系」なんて言っていない

慣性系において

ma = F

が成り立つ。この式でF = 0としたら

a = 0

か? 早く答えろよ
0976ご冗談でしょう?名無しさん
垢版 |
2021/12/06(月) 19:04:15.52ID:???
貢献したくても議論にさえなってないんだが?
それともこれが議論のつもりなの?
言いたくなくても馬鹿としか言えん
0983ご冗談でしょう?名無しさん
垢版 |
2021/12/07(火) 11:32:02.44ID:???
>>982
「ma=F」が成り立つのは慣性系において。
第一法則は慣性系の存在を主張している。
だから第一法則は第二法則の系ではない。
0984ご冗談でしょう?名無しさん
垢版 |
2021/12/07(火) 11:34:30.06ID:???
>>982
「ma=F」が成り立つのは慣性系において。
第一法則は慣性系の存在を主張している。
だから第一法則は第二法則の系ではない。
0985ご冗談でしょう?名無しさん
垢版 |
2021/12/07(火) 11:34:39.04ID:???
>>982
「ma=F」が成り立つのは慣性系において。
第一法則は慣性系の存在を主張している。
だから第一法則は第二法則の系ではない。
0986ご冗談でしょう?名無しさん
垢版 |
2021/12/07(火) 12:48:19.89ID:wbI8XsfU
>>983-985
何度も言うように、そんなことは聞いていない

慣性系において「ma=F」が成り立つ
この式でF=0を代入したら、a=0か?

早く答えてくれ
0987ご冗談でしょう?名無しさん
垢版 |
2021/12/07(火) 13:31:31.97ID:???
>>986
だから「ma=F」が成り立つのは慣性系においてなんだよ。
第一法則は慣性系の存在を保証しているのであって、第二法則から導かれるのではない。
0991ご冗談でしょう?名無しさん
垢版 |
2021/12/07(火) 14:52:48.50ID:???
>>990
誰も「慣性系の存在が第二法則の系」なんて言っていない

慣性系において

ma = F

が成り立つ。この式でF = 0としたら

a = 0

か?いい加減、答えてくれ。
0992ご冗談でしょう?名無しさん
垢版 |
2021/12/07(火) 16:04:56.52ID:???
yes/noで答える質問から逃げるのはつまりそういうことよ
もうすぐスレが終わるので>>983-985のように連投してまで埋めようとするw
負け戦をなんとか有耶無耶にするためにw
10011001
垢版 |
Over 1000Thread
このスレッドは1000を超えました。
新しいスレッドを立ててください。
life time: 112日 20時間 44分 50秒
10021002
垢版 |
Over 1000Thread
5ちゃんねるの運営はプレミアム会員の皆さまに支えられています。
運営にご協力お願いいたします。


───────────────────
《プレミアム会員の主な特典》
★ 5ちゃんねる専用ブラウザからの広告除去
★ 5ちゃんねるの過去ログを取得
★ 書き込み規制の緩和
───────────────────

会員登録には個人情報は一切必要ありません。
月300円から匿名でご購入いただけます。

▼ プレミアム会員登録はこちら ▼
https://premium.5ch.net/

▼ 浪人ログインはこちら ▼
https://login.5ch.net/login.php
レス数が1000を超えています。これ以上書き込みはできません。

ニューススポーツなんでも実況